PT30.S2.Q20 - Scientist: Some critics of public funding

lizpillizpil Member
edited August 2018 in Logical Reasoning 282 karma

Hola 7Sagers,

I'm super struggling with this question simply because I feel like the answer makes a big jump. It almost seem like its a sufficient assumption question. I just don't see where the stimulus indicates where the public support in question indicates there was not any indication of public benefit.

Can anyone help?

Thank you!

Admin note: edited title
https://7sage.com/lsat_explanations/lsat-30-section-2-question-20/

Comments

  • BinghamtonDaveBinghamtonDave Alum Member 🍌🍌
    8694 karma

    This is a complex argument structure, and is a question where we see a confluence of several of the lessons in the CC.

    In a very abstract form we are told that other people say that B is necessary for A. Namely:
    If continued funding then can be indicated how the public would benefit

    The argument here makes a rare move, the argument says "if the critics are correct" This turn from other peoples opinion to our main argument is telling us two things:
    1.Correct means that "can be indicated how the public would benefit" is necessary for "continued funding. Correct here means that the whole conditional holds.
    2.The "If" tells us that that conditional holding is sufficient for some state of events. So if that conditional holds, we should be expected to see some other state of events.

    So, our author is telling us that if the other people's opinion (which is a conditional relationship) holds, then we should see a specific state of events. What is that state of events we should see? That there would not be tremendous public support.

    So lets pause here an review what it is we have in front of us: Our author has stated:
    If (Continued fund----->can be indicated how the public would benefit) if that conditional holds in the world Then we should see not tremendous public support.

    Here is the next layer of this problem: our author has given us this complex conditional:
    if this conditional relationship holds then we should not see tremendous public support.
    But then our author says: in the real world we do indeed see tremendous public support. This effective fails the state of events required for the other people's opinion to hold. We are allowed to say at this juncture that the MBT of this question is:
    other people's opinion (which is a conditional statement), does not hold.

    Now, here is the 5th and final layer to this question: how do we say that the relationship: If continued funding then can be indicated how the public would benefit
    does not hold?

    We say that the way (E) describes it.

    I have thought of a potential analog to this problem:
    some people say that every house cat is kind. But if this were true, we would never expect to see a house cat holding a family hostage in Florida. But in fact we do see such a case.
    Therefore, other people's opinion (which is a conditional relationship) is wrong.

    I hope this helps
    David

  • lizpillizpil Member
    282 karma

    David,

    God bless you, seriously. This is a great explanation with a nice analogy kicker. I'm still stuck on this. The public can support something without knowing how it can help them. That to me is different than saying that the program is justified when the public supports them. Where do we link public support and justification?

  • OhnoeshalpmeOhnoeshalpme Alum Member
    edited August 2018 2531 karma

    To make it simpler, we can see the argument as... "If what you're saying is true, then we would not be seeing what we're seeing. Since we are in fact seeing this thing, we have to conclude that what you are saying is not entirely true."

    What is stated as true is that "public benefit is necessary for justification of spending" and the author is saying, "if it were the case that public benefit was necessary for a justification of spending than we wouldn't be seeing this public support." since we are in a world where public support exists it must be the case that critics were not right.

    Part of the confusion here I think is that the argument makes very little worldly sense. But it would be 100% equally valid if, instead of public support, we put in "widespread skydiving". They want you to waste time trying to figure out why these two things are incompatible but we just have to accept that what the author is saying. This is why breaking things down into lawgic can be so useful sometimes because it helps us to see the structure underlying the argument and detaches us from the unnecessary and intentionally distracting details in the stimulus.

  • lizpillizpil Member
    282 karma

    I agree. There's lots of public support for things that don't benefit the public. Ha ha

Sign In or Register to comment.